subject
Mathematics, 23.09.2019 11:30 dayanara72

Indirect proof ? stuck between last 2 choices
what is the assumption needed to begin an indirect proof of the following statement?
"a team is as strong as its weakest member."
a. a team is as strong as its weakest member.
b. a team is as strong as its strongest member.
c. a team is not as strong as its weakest member.
d. a team is not as strong as its strongest member.

ansver
Answers: 1

Another question on Mathematics

question
Mathematics, 20.06.2019 18:02
What is the equation of a parabola y(x) that has a vertex at point (− 1/3 , 0.3) and passes through point (− 2/15 , − 1/2 ).
Answers: 1
question
Mathematics, 21.06.2019 19:00
Billy plotted −3 4 and −1 4 on a number line to determine that −3 4 is smaller than −1 4 is he correct? explain why or why not?
Answers: 3
question
Mathematics, 21.06.2019 19:10
1jessica's home town is a mid-sized city experiencing a decline in population. the following graph models the estimated population if the decline continues at the same rate. select the most appropriate unit for the measure of time that the graph represents. a. years b. hours c. days d. weeks
Answers: 2
question
Mathematics, 21.06.2019 21:00
What is the unit rate of, 75% high fiber chimp food to 25% high protein chimp food.
Answers: 1
You know the right answer?
Indirect proof ? stuck between last 2 choices
what is the assumption needed to begin an indir...
Questions
question
Mathematics, 08.07.2019 05:40
question
Mathematics, 08.07.2019 05:40
question
Chemistry, 08.07.2019 05:40
question
Business, 08.07.2019 05:40
Questions on the website: 13722363